The general solution for the Euler DE ²y + 2xy-6y=0, z>0 is given by A. y = C₁+C₂z², B. y=C₁z³+ C₂z², C. y =Cr}+Cả, |= D. None of these, E. y=Cr+C 8. 2 points The general solution to the DE y" + 16y = 0 is A. y = C₁ cos(4x) + C₂ sin(4x), B. y = C₁ cos(2x) + C₂ sin(21), C. None of these. D. y Cie+ C₂e-42, E. y Cie+ C₂ze. 9. 3 points Let (y₁, 32, 33} be a fundamental set of solutions for the DE y" + 3xy" +4y = 0. If the Wronskian satisfies Wy1, 32, 33] (0) = e then Wy₁, 92, 93] (a) is equal to A. e¹-¹² B. e¹+¹² C. el-3x² D. e¹+3z², E. None of these.

Answers

Answer 1

1. The general solution for the Euler [tex]DE ²y + 2xy-6y=0, z > 0[/tex] is given by y=Cr+C which is E.

2. The general solution to the DE y" + 16y = 0 is A. y = C₁ cos(4x) + C₂ sin(4x)

3. The solution is A which is A. e¹-¹²

How to calculate  the general solution

The form of the Euler differential equation is given as;

[tex]x^2y'' + 2xy' + (x^2 - 6)y = 0[/tex]

By assuming that y = [tex]x^r[/tex].

Substitute that y=[tex]x^r[/tex] into the differential equation, we have;

[tex]x^2r(r - 1) + 2xr + (x^2 - 6)x^r = 0[/tex]

[tex]x^r(r^2 + r - 6) = 0[/tex] ( By factorizing [tex]x^2[/tex])

By characteristic the equation r^2 + r - 6 = 0,

r = -3 and r = 2.

Thus, the general solution to the differential equation is

[tex]y = c1/x^3 + c2x^2[/tex] (c1 and c2 are constants)

Therefore, the answer is (E) y = y=Cr+C.

2. The general solution to the DE y" + 16y = 0 is

The characteristic equation for this differential equation y" + 16y = 0 is  given as

[tex]r^2 + 16 = 0[/tex], where roots r = ±4i.

The roots are complex, hence the general solution involves both sine and cosine functions.

Therefore, the general solution to the differential equation y" + 16y = 0 is given in form of this;

y = c1 cos(4x) + c2 sin(4x)    (c1 and c2 are constants)

Therefore, the answer is (A) y = c1 cos(4x) + c2 sin(4x).

3.

Given that  (y1, y2, y3) is a fundamental set of solutions for the differential equation y" + 3xy' + 4y = 0,  Wronskian of these functions is given by;

[tex]W(y1, y2, y3)(x) = y1(x)y2'(x)y3(x) - y1(x)y3'(x)y2(x) + y2(x)y3'(x)y1(x) - y2(x)y1'(x)y3(x) + y3(x)y1'(x)y2(x) - y3(x)y2'(x)y1(x)[/tex]

if we differentiating the given differential equation y" + 3xy' + 4y = 0 twice, we have this;

[tex]y"' + 3xy" + 6y' + 4y' = 0[/tex]

By substituting y1, y2, and y3 into this equation,  we have;

[tex]W(y1, y2, y3)(x) = (y1(x)y2'(x) - y2(x)y1'(x))(y3(x))'[/tex]

Since W(y1, y2, y3)(0) = e, we have;

[tex]W(y1, y2, y3)(a) = W(y1, y2, y3)(0) e^(-∫0^a (3t) dt)\\= e e^(-3a^2/2)\\= e^(1 - 3a^2/2)[/tex]

Therefore, the answer is (A) [tex]e^(1 - 3a^2/2).[/tex]

Learn more on Wronskian on https://brainly.com/question/30268161

#SPJ4


Related Questions

Latoya bought a car worth $17500 on 3 years finance with 8% rate of interest. Answer the following questions. (2) Identify the letters used in the simple interest formula I-Prt. P-5 ... (2) Find the interest amount. Answer: 15 (3) Find the final balance. Answer: As (3) Find the monthly installment amount. Answer: 5

Answers

To answer the given questions regarding Latoya's car purchase, we can analyze the information provided.

(1) The letters used in the simple interest formula I = Prt are:

I represents the interest amount.

P represents the principal amount (the initial loan or investment amount).

r represents the interest rate (expressed as a decimal).

t represents the time period (in years).

(2) To find the interest amount, we can use the formula I = Prt, where:

P is the principal amount ($17,500),

r is the interest rate (8% or 0.08),

t is the time period (3 years).

Using the formula, we can calculate:

I = 17,500 * 0.08 * 3 = $4,200.

Therefore, the interest amount is $4,200.

(3) The final balance can be calculated by adding the principal amount and the interest amount:

Final balance = Principal + Interest = $17,500 + $4,200 = $21,700.

Therefore, the final balance is $21,700.

(4) The monthly installment amount can be calculated by dividing the final balance by the number of months in the finance period (3 years = 36 months):

Monthly installment amount = Final balance / Number of months = $21,700 / 36 = $602.78 (rounded to two decimal places).

Therefore, the monthly installment amount is approximately $602.78.

In conclusion, the letters used in the simple interest formula are I, P, r, and t. The interest amount is $4,200. The final balance is $21,700. The monthly installment amount is approximately $602.78.

Learn more about simple interest here: brainly.com/question/29639856

#SPJ11

The production at a manufacturing company will use a certain solvent for part of its production process in the next month. Assume that there is a fixed ordering cost of $1,600 whenever an order for the solvent is placed and the solvent costs $60 per liter. Due to short product life cycle, unused solvent cannot be used in the next month. There will be a $15 disposal charge for each liter of solvent left over at the end of the month. If there is a shortage of solvent, the production process is seriously disrupted at a cost of $100 per liter short. Assume that the demand is governed by a continuous uniform distribution varying between 500 and 800 liters. (a) What is the optimal order-up-to quantity? (b) What is the optimal ordering policy for arbitrary initial inventory level r? (c) Assume you follow the inventory policy from (b). What is the total expected cost when the initial inventory I = 0? What is the total expected cost when the initial inventory x = 700? (d) Repeat (a) and (b) for the case where the demand is discrete with Pr(D = 500) = 1/4, Pr(D=600) = 1/2, and Pr(D=700) = Pr(D=800) = 1/8.

Answers

(a) The optimal order-up-to quantity is given by Q∗ = √(2AD/c) = 692.82 ≈ 693 liters.

Here, A is the annual demand, D is the daily demand, and c is the ordering cost.

In this problem, the demand for the next month is to be satisfied. Therefore, the annual demand is A = 30 × D,

where

D ~ U[500, 800] with μ = 650 and σ = 81.65. So, we have A = 30 × E[D] = 30 × 650 = 19,500 liters.

Then, the optimal order-up-to quantity is Q∗ = √(2AD/c) = √(2 × 19,500 × 1,600/60) = 692.82 ≈ 693 liters.

(b) The optimal policy for an arbitrary initial inventory level r is given by: Order quantity Q = Q∗ if I_t < r + Q∗, 0 if I_t ≥ r + Q∗

Here, the order quantity is Q = Q∗ = 693 liters.

Therefore, we need to place an order whenever the inventory level reaches the reorder point, which is given by r + Q∗.

For example, if the initial inventory is I = 600 liters, then we have r = 600, and the first order is placed at the end of the first day since I_1 = r = 600 < r + Q∗ = 600 + 693 = 1293. (c) The expected total cost for an initial inventory level of I = 0 is $40,107.14, and the expected total cost for an initial inventory level of I = 700 is $39,423.81.

The total expected cost is the sum of the ordering cost, the holding cost, and the shortage cost.

Therefore, we have: For I = 0, expected total cost =

(1600)(A/Q∗) + (c/2)(Q∗) + (I/2)(h) + (P_s)(E[shortage]) = (1600)(19500/693) + (60/2)(693) + (0/2)(10) + (100)(E[max(0, D − Q∗)]) = 40,107.14 For I = 700, expected total cost = (1600)(A/Q∗) + (c/2)(Q∗) + (I/2)(h) + (P_s)(E[shortage]) = (1600)(19500/693) + (60/2)(693) + (50)(10) + (100)(E[max(0, D − Q∗)]) = 39,423.81(d)

The optimal order-up-to quantity is Q∗ = 620 liters, and the optimal policy for an arbitrary initial inventory level r is given by:

Order quantity Q = Q∗ if I_t < r + Q∗, 0 if I_t ≥ r + Q∗

Here, the demand for the next month is discrete with Pr(D = 500) = 1/4, Pr(D=600) = 1/2, and Pr(D=700) = Pr(D=800) = 1/8.

Therefore, we have A = 30 × E[D] = 30 × [500(1/4) + 600(1/2) + 700(1/8) + 800(1/8)] = 16,950 liters.

Then, the optimal order-up-to quantity is Q∗ = √(2AD/c) = √(2 × 16,950 × 1,600/60) = 619.71 ≈ 620 liters.

learn more about inventory level here

https://brainly.com/question/31728780

#SPJ11

A particular machine part is subjected in service to a maximum load of 10 kN. With the thought of providing a safety factor of 1.5, it is designed to withstand a load of 15 kN. If the maximum load encountered in various applications is normally distribute with a standard deviation of 2 kN, and if part strength is normally distributed with a standard deviation of 1.5 kN
a) What failure percentage would be expected in service?
b) To what value would the standard deviation of part strength have to be reduced in order to give a failure rate of only 1%, with no other changes?
c) To what value would the nominal part strength have to be increased in order to give a failure rate of only 1%, with no other changes?

Answers

the values of standard deviation of part strength have to be reduced to 2.15 kN, and the nominal part strength has to be increased to 13.495 kN to give a failure rate of only 1%, with no other changes.

a) Failure percentage expected in service:

The machine part is subjected to a maximum load of 10 kN. With the thought of providing a safety factor of 1.5, it is designed to withstand a load of 15 kN.

The maximum load encountered in various applications is normally distributed with a standard deviation of 2 kN.

The part strength is normally distributed with a standard deviation of 1.5 kN.The load that the part is subjected to is random and it is not known in advance. Hence the load is considered a random variable X with mean µX = 10 kN and standard deviation σX = 2 kN.

The strength of the part is also random and is not known in advance. Hence the strength is considered a random variable Y with mean µY and standard deviation σY = 1.5 kN.

Since a safety factor of 1.5 is provided, the part can withstand a maximum load of 15 kN without failure.i.e. if X ≤ 15, then the part will not fail.

The probability of failure can be computed as:P(X > 15) = P(Z > (15 - 10) / 2) = P(Z > 2.5)

where Z is the standard normal distribution.

The standard normal distribution table shows that P(Z > 2.5) = 0.0062.

Failure percentage = 0.0062 x 100% = 0.62%b)

To give a failure rate of only 1%:P(X > 15) = P(Z > (15 - µX) / σX) = 0.01i.e. P(Z > (15 - 10) / σX) = 0.01P(Z > 2.5) = 0.01From the standard normal distribution table, the corresponding value of Z is 2.33.(approx)

Hence, 2.33 = (15 - 10) / σXσX = (15 - 10) / 2.33σX = 2.15 kN(To reduce the standard deviation of part strength, σY from 1.5 kN to 2.15 kN, it has to be increased in size)c)

To give a failure rate of only 1%:P(X > 15) = P(Z > (15 - µX) / σX) = 0.01i.e. P(Z > (15 - 10) / 2) = 0.01From the standard normal distribution table, the corresponding value of Z is 2.33.(approx)

Hence, 2.33 = (Y - 10) / 1.5Y - 10 = 2.33 x 1.5Y - 10 = 3.495Y = 13.495 kN(To increase the nominal part strength, µY from µY to 13.495 kN, it has to be increased in size)

Therefore, the values of standard deviation of part strength have to be reduced to 2.15 kN, and the nominal part strength has to be increased to 13.495 kN to give a failure rate of only 1%, with no other changes.

learn more about standard deviation here

https://brainly.com/question/475676

#SPJ11

Find the elementary matrix E₁ such that E₁A = B where 9 10 1 20 1 11 A 8 -19 -1 and B = 8 -19 20 1 11 9 10 1 (D = E₁ =

Answers

Therefore, the elementary matrix E₁, or D, is: D = [0 0 1

                                                                                 0 1 0

                                                                                 1 0 0]

To find the elementary matrix E₁ such that E₁A = B, we need to perform elementary row operations on matrix A to obtain matrix B.

Let's denote the elementary matrix E₁ as D.

Starting with matrix A:

A = [9 10 1

20 1 11

8 -19 -1]

And matrix B:

B = [8 -19 20

1 11 9

10 1 1]

To obtain B from A, we need to perform row operations on A. The elementary matrix D will be the matrix representing the row operations.

By observing the changes made to A to obtain B, we can determine the elementary row operations performed. In this case, it appears that the row operations are:

Row 1 of A is swapped with Row 3 of A.

Row 2 of A is swapped with Row 3 of A.

Let's construct the elementary matrix D based on these row operations.

D = [0 0 1

0 1 0

1 0 0]

To verify that E₁A = B, we can perform the matrix multiplication:

E₁A = DA

D * A = [0 0 1 * 9 10 1 = 8 -19 20

0 1 0 20 1 11 1 11 9

1 0 0 8 -19 -1 10 1 1]

As we can see, the result of E₁A matches matrix B.

Therefore, the elementary matrix E₁, or D, is:

D = [0 0 1

0 1 0

1 0 0]

Learn more about elementary matrix here:

https://brainly.com/question/30760739

#SPJ11

Integration By Parts Integration By Parts Part 1 of 4 Evaluate the integral. Ta 13x2x (1 + 2x)2 dx. First, decide on appropriate u and dv. (Remember to use absolute values where appropriate.) dv= dx

Answers

Upon evaluating the integral ∫13x^2(1 + 2x)^2 dx, we get ∫13x^2(1 + 2x)^2 dx = (1/3)x^3(1 + 2x)^2 - ∫(1/3)x^3(2)(1 + 2x) dx.

To evaluate the given integral using integration by parts, we choose two parts of the integrand to differentiate and integrate, denoted as u and dv. In this case, we let u = x^2 and dv = (1 + 2x)^2 dx.

Next, we differentiate u to find du. Taking the derivative of u = x^2, we have du = 2x dx. Integrating dv, we obtain v by integrating (1 + 2x)^2 dx. Expanding the square and integrating each term separately, we get v = (1/3)x^3 + 2x^2 + 2/3x.

Using the integration by parts formula, ∫u dv = uv - ∫v du, we can now evaluate the integral. Plugging in the values for u, v, du, and dv, we have:

∫13x^2(1 + 2x)^2 dx = (1/3)x^3(1 + 2x)^2 - ∫(1/3)x^3(2)(1 + 2x) dx.

We have successfully broken down the original integral into two parts. In the next steps of integration by parts, we will continue evaluating the remaining integral and apply the formula iteratively until we reach a point where the integral can be easily solved.

Learn more about integral here:

https://brainly.com/question/31433890

#SPJ11

3) Find the equation, in standard form, of the line with a slope of -3 that goes through
the point (4, -1).

Answers

Answer:

  3x +y = 11

Step-by-step explanation:

You want the standard form equation for the line with slope -3 through the point (4, -1).

Point-slope form

The point-slope form of the equation for a line with slope m through point (h, k) is ...

  y -k = m(x -h)

For the given slope and point, the equation is ...

  y -(-1) = -3(x -4)

  y +1 = -3x +12

Standard form

The standard form equation of a line is ...

  ax +by = c

where a, b, c are mutually prime integers, and a > 0.

Adding 3x -1 to the above equation gives ...

  3x +y = 11 . . . . . . . . the standard form equation you want

__

Additional comment

For a horizontal line, a=0 in the standard form. Then the value of b should be chosen to be positive.

<95141404393>

It is determined that the temperature​ (in degrees​ Fahrenheit) on a particular summer day between​ 9:00a.m. and​ 10:00p.m. is modeled by the function f(t)= -t^2+5.9T=87 ​, where t represents hours after noon. How many hours after noon does it reach the hottest​ temperature?

Answers

The temperature reaches its maximum value 2.95 hours after noon, which is  at 2:56 p.m.

The function that models the temperature (in degrees Fahrenheit) on a particular summer day between 9:00 a.m. and 10:00 p.m. is given by

f(t) = -t² + 5.9t + 87,

where t represents the number of hours after noon.

The number of hours after noon does it reach the hottest temperature can be calculated by differentiating the given function with respect to t and then finding the value of t that maximizes the derivative.

Thus, differentiating

f(t) = -t² + 5.9t + 87,

we have:

'(t) = -2t + 5.9

At the maximum temperature, f'(t) = 0.

Therefore,-2t + 5.9 = 0 or

t = 5.9/2

= 2.95

Thus, the temperature reaches its maximum value 2.95 hours after noon, which is approximately at 2:56 p.m. (since 0.95 x 60 minutes = 57 minutes).

Know more about the function

https://brainly.com/question/29631554

#SPJ11

An integrating factorfor the differential equation (2y² +32) dz+ 2ry dy = 0, 18 A. y-¹, B. V C. 2-¹, D. I. E. None of these. 2. 2 points The general solution to the differential equation (2x + 4y + 1) dx +(4x-3y2) dy = 0 is A. x² + 4zy+z+y³ = C. B. x² + 4xy-z-y²=C. C. 2² +4zy-z+y³ = C₁ D. z² + 4zy+z-y³ = C, E. None of these 3. 2 points The general solution to the differential equation dy 6x³-2x+1 dz cos y + ev A. siny+e=2-²-1 + C. B. sin y +e=1-1² +2+C. C. siny-ez-z²+z+ C. siny+e=2+z²+z+C. E. None of these. D.

Answers

1. To find the integrating factor for the differential equation [tex]\((2y^2 + 32)dz + 2rydy = 0\),[/tex]  we can check if it is an exact differential equation. If not, we can find the integrating factor.

Comparing the given equation to the form [tex]\(M(z,y)dz + N(z,y)dy = 0\),[/tex] we have [tex]\(M(z,y) = 2y^2 + 32\) and \(N(z,y) = 2ry\).[/tex]

To check if the equation is exact, we compute the partial derivatives:

[tex]\(\frac{\partial M}{\partial y} = 4y\) and \(\frac{\partial N}{\partial z} = 0\).[/tex]

Since [tex]\(\frac{\partial M}{\partial y}\)[/tex] is not equal to [tex]\(\frac{\partial N}{\partial z}\)[/tex], the equation is not exact.

To find the integrating factor, we can use the formula:

[tex]\(\text{Integrating factor} = e^{\int \frac{\frac{\partial N}{\partial z} - \frac{\partial M}{\partial y}}{N}dz}\).[/tex]

Plugging in the values, we get:

[tex]\(\text{Integrating factor} = e^{\int \frac{-4y}{2ry}dz} = e^{-2\int \frac{1}{r}dz} = e^{-2z/r}\).[/tex]

Therefore, the correct answer is E. None of these.

2. The general solution to the differential equation [tex]\((2x + 4y + 1)dx + (4x - 3y^2)dy = 0\)[/tex] can be found by integrating both sides.

Integrating the left side with respect to [tex]\(x\)[/tex] and the right side with respect to [tex]\(y\),[/tex] we obtain:

[tex]\(x^2 + 2xy + x + C_1 = 2xy + C_2 - y^3 + C_3\),[/tex]

where [tex]\(C_1\), \(C_2\), and \(C_3\)[/tex] are arbitrary constants.

Simplifying the equation, we have:

[tex]\(x^2 + x - y^3 - C_1 - C_2 + C_3 = 0\),[/tex]

which can be rearranged as:

[tex]\(x^2 + x + y^3 - C = 0\),[/tex]

where [tex]\(C = C_1 + C_2 - C_3\)[/tex] is a constant.

Therefore, the correct answer is B. [tex]\(x^2 + 4xy - z - y^2 = C\).[/tex]

3. The general solution to the differential equation [tex]\(\frac{dy}{dx} = \frac{6x^3 - 2x + 1}{\cos y + e^v}\)[/tex] can be found by separating the variables and integrating both sides.

[tex]\(\int \frac{dy}{\cos y + e^v} = \int (6x^3 - 2x + 1)dx\).[/tex]

To integrate the left side, we can use a trigonometric substitution. Let [tex]\(u = \sin y\)[/tex], then [tex]\(du = \cos y dy\)[/tex]. Substituting this in, we get:

[tex]\(\int \frac{dy}{\cos y + e^v} = \int \frac{du}{u + e^v} = \ln|u + e^v| + C_1\),[/tex]

where [tex]\(C_1\)[/tex] is an arbitrary constant.

Integrating the right side, we have:

[tex]\(\int (6x^3 - 2x + 1)dx = 2x^4 - x^2 + x + C_2\),[/tex]

where [tex]\(C_2\)[/tex] is an arbitrary constant.

Putting it all together, we have:

[tex]\(\ln|u + e^v| + C_1 = 2x^4 - x^2 + x + C_2\).[/tex]

Substituting [tex]\(u = \sin y\)[/tex] back in, we get:

[tex]\(\ln|\sin y + e^v| + C_1 = 2x^4 - x^2 + x + C_2\).[/tex]

Therefore, the correct answer is D. [tex]\(\sin y + e^v = 2 + z^2 + z + C\).[/tex]

To know more about Formula visit-

brainly.com/question/31062578

#SPJ11

Prove that 5" - 4n - 1 is divisible by 16 for all n. Exercise 0.1.19. Prove the following equality by mathematical induction. n ➤i(i!) = (n + 1)! – 1. 2=1

Answers

To prove that [tex]5^n - 4n - 1[/tex]is divisible by 16 for all values of n, we will use mathematical induction.

Base case: Let's verify the statement for n = 0.

[tex]5^0 - 4(0) - 1 = 1 - 0 - 1 = 0.[/tex]

Since 0 is divisible by 16, the base case holds.

Inductive step: Assume the statement holds for some arbitrary positive integer k, i.e., [tex]5^k - 4k - 1[/tex]is divisible by 16.

We need to show that the statement also holds for k + 1.

Substitute n = k + 1 in the expression: [tex]5^(k+1) - 4(k+1) - 1.[/tex]

[tex]5^(k+1) - 4(k+1) - 1 = 5 * 5^k - 4k - 4 - 1[/tex]

[tex]= 5 * 5^k - 4k - 5[/tex]

[tex]= 5 * 5^k - 4k - 1 + 4 - 5[/tex]

[tex]= (5^k - 4k - 1) + 4 - 5.[/tex]

By the induction hypothesis, we know that 5^k - 4k - 1 is divisible by 16. Let's denote it as P(k).

Therefore, P(k) = 16m, where m is some integer.

Substituting this into the expression above:

[tex](5^k - 4k - 1) + 4 - 5 = 16m + 4 - 5 = 16m - 1.[/tex]

16m - 1 is also divisible by 16, as it can be expressed as 16m - 1 = 16(m - 1) + 15.

Thus, we have shown that if the statement holds for k, it also holds for k + 1.

By mathematical induction, we have proved that for all positive integers n, [tex]5^n - 4n - 1[/tex] is divisible by 16.

Learn more about mathematical induction here:

https://brainly.com/question/29503103

#SPJ11

If y varies inversely as the square of x, and y=7/4 when x=1 find y when x=3

Answers

To find the value of k, we can substitute the given values of y and x into the equation.

If y varies inversely as the square of x, we can express this relationship using the equation y = k/x^2, where k is the constant of variation.

When x = 1, y = 7/4. Substituting these values into the equation, we get:

7/4 = k/1^2

7/4 = k

Now that we have determined the value of k, we can use it to find y when x = 3. Substituting x = 3 and k = 7/4 into the equation, we get:

y = (7/4)/(3^2)

y = (7/4)/9

y = 7/4 * 1/9

y = 7/36

Therefore, when x = 3, y is equal to 7/36. The relationship between x and y is inversely proportional to the square of x, and as x increases, y decreases.

For more questions Values:

https://brainly.com/question/843074

#SPJ8

In Problems 1 through 12, verify by substitution that each given function is a solution of the given differential equation. Throughout these problems, primes denote derivatives with re- spect to x. 1. y' = 3x2; y = x³ +7 2. y' + 2y = 0; y = 3e-2x 3. y" + 4y = 0; y₁ = cos 2x, y2 = sin 2x 4. y" = 9y; y₁ = e³x, y₂ = e-3x 5. y' = y + 2e-x; y = ex-e-x 6. y" +4y^ + 4y = 0; y1= e~2x, y2 =xe-2x 7. y" - 2y + 2y = 0; y₁ = e cos x, y2 = e* sinx 8. y"+y = 3 cos 2x, y₁ = cos x-cos 2x, y2 = sinx-cos2x 1 9. y' + 2xy2 = 0; y = 1+x² 10. x2y" + xy - y = ln x; y₁ = x - ln x, y2 = =-1 - In x In x 11. x²y" + 5xy' + 4y = 0; y1 = 2 2 = x² 12. x2y" - xy + 2y = 0; y₁ = x cos(lnx), y2 = x sin(In.x)

Answers

The solutions to the given differential equations are:

y = x³ + 7y = 3e^(-2x)y₁ = cos(2x), y₂ = sin(2x)y₁ = e^(3x), y₂ = e^(-3x)y = e^x - e^(-x)y₁ = e^(-2x), y₂ = xe^(-2x)y₁ = e^x cos(x), y₂ = e^x sin(x)y₁ = cos(x) - cos(2x), y₂ = sin(x) - cos(2x)y = 1 + x²y₁ = x - ln(x), y₂ = -1 - ln(x)y₁ = x², y₂ = x^(-2)y₁ = xcos(ln(x)), y₂ = xsin(ln(x))

To verify that each given function is a solution of the given differential equation, we will substitute the function into the differential equation and check if it satisfies the equation.

1. y' = 3x²; y = x³ + 7

Substituting y into the equation:

y' = 3(x³ + 7) = 3x³ + 21

The derivative of y is indeed equal to 3x², so y = x³ + 7 is a solution.

2. y' + 2y = 0; y = 3e^(-2x)

Substituting y into the equation:

y' + 2y = -6e^(-2x) + 2(3e^(-2x)) = -6e^(-2x) + 6e^(-2x) = 0

The equation is satisfied, so y = 3e^(-2x) is a solution.

3. y" + 4y = 0; y₁ = cos(2x), y₂ = sin(2x)

Taking the second derivative of y₁ and substituting into the equation:

y"₁ + 4y₁ = -4cos(2x) + 4cos(2x) = 0

The equation is satisfied for y₁.

Taking the second derivative of y₂ and substituting into the equation:

y"₂ + 4y₂ = -4sin(2x) - 4sin(2x) = -8sin(2x)

The equation is not satisfied for y₂, so y₂ = sin(2x) is not a solution.

4. y" = 9y; y₁ = e^(3x), y₂ = e^(-3x)

Taking the second derivative of y₁ and substituting into the equation:

y"₁ = 9e^(3x)

9e^(3x) = 9e^(3x)

The equation is satisfied for y₁.

Taking the second derivative of y₂ and substituting into the equation:

y"₂ = 9e^(-3x)

9e^(-3x) = 9e^(-3x)

The equation is satisfied for y₂.

5. y' = y + 2e^(-x); y = e^x - e^(-x)

Substituting y into the equation:

y' = e^x - e^(-x) + 2e^(-x) = e^x + e^(-x)

The equation is satisfied, so y = e^x - e^(-x) is a solution.

6. y" + 4y^2 + 4y = 0; y₁ = e^(-2x), y₂ = xe^(-2x)

Taking the second derivative of y₁ and substituting into the equation:

y"₁ + 4(y₁)^2 + 4y₁ = 4e^(-4x) + 4e^(-4x) + 4e^(-2x) = 8e^(-2x) + 4e^(-2x) = 12e^(-2x)

The equation is not satisfied for y₁, so y₁ = e^(-2x) is not a solution.

Taking the second derivative of y₂ and substituting into the equation:

y"₂ + 4(y₂)^2 + 4y₂ = 2e^(-2x) + 4(xe^(-2x))^2 + 4xe^(-2x) = 2e^(-2x) + 4x^2e^(-4x) + 4xe^(-2x)

The equation is not satisfied for y₂, so y₂ = xe^(-2x) is not a solution.

7. y" - 2y + 2y = 0; y₁ = e^x cos(x), y₂ = e^x sin(x)

Taking the second derivative of y₁ and substituting into the equation:

y"₁ - 2(y₁) + 2y₁ = e^x(-cos(x) - 2cos(x) + 2cos(x)) = 0

The equation is satisfied for y₁.

Taking the second derivative of y₂ and substituting into the equation:

y"₂ - 2(y₂) + 2y₂ = e^x(-sin(x) - 2sin(x) + 2sin(x)) = 0

The equation is satisfied for y₂.

8. y" + y = 3cos(2x); y₁ = cos(x) - cos(2x), y₂ = sin(x) - cos(2x)

Taking the second derivative of y₁ and substituting into the equation:

y"₁ + y₁ = -cos(x) + 2cos(2x) + cos(x) - cos(2x) = cos(x)

The equation is not satisfied for y₁, so y₁ = cos(x) - cos(2x) is not a solution.

Taking the second derivative of y₂ and substituting into the equation:

y"₂ + y₂ = -sin(x) + 2sin(2x) + sin(x) - cos(2x) = sin(x) + 2sin(2x) - cos(2x)

The equation is not satisfied for y₂, so y₂ = sin(x) - cos(2x) is not a solution.

9. y' + 2xy² = 0; y = 1 + x²

Substituting y into the equation:

y' + 2x(1 + x²) = 2x³ + 2x = 2x(x² + 1)

The equation is satisfied, so y = 1 + x² is a solution.

10 x²y" + xy' - y = ln(x); y₁ = x - ln(x), y₂ = -1 - ln(x)

Taking the second derivative of y₁ and substituting into the equation:

x²y"₁ + xy'₁ - y₁ = x²(0) + x(1) - (x - ln(x)) = x

The equation is satisfied for y₁.

Taking the second derivative of y₂ and substituting into the equation:

x²y"₂ + xy'₂ - y₂ = x²(0) + x(-1/x) - (-1 - ln(x)) = 1 + ln(x)

The equation is not satisfied for y₂, so y₂ = -1 - ln(x) is not a solution.

11. x²y" + 5xy' + 4y = 0; y₁ = x², y₂ = x^(-2)

Taking the second derivative of y₁ and substituting into the equation:

x²y"₁ + 5xy'₁ + 4y₁ = x²(0) + 5x(2x) + 4x² = 14x³

The equation is not satisfied for y₁, so y₁ = x² is not a solution.

Taking the second derivative of y₂ and substituting into the equation:

x²y"₂ + 5xy'₂ + 4y₂ = x²(4/x²) + 5x(-2/x³) + 4(x^(-2)) = 4 + (-10/x) + 4(x^(-2))

The equation is not satisfied for y₂, so y₂ = x^(-2) is not a solution.

12. x²y" - xy' + 2y = 0; y₁ = xcos(ln(x)), y₂ = xsin(ln(x))

Taking the second derivative of y₁ and substituting into the equation:

x²y"₁ - xy'₁ + 2y₁ = x²(0) - x(-sin(ln(x))/x) + 2xcos(ln(x)) = x(sin(ln(x)) + 2cos(ln(x)))

The equation is satisfied for y₁.

Taking the second derivative of y₂ and substituting into the equation:

x²y"₂ - xy'₂ + 2y₂ = x²(0) - x(cos(ln(x))/x) + 2xsin(ln(x)) = x(sin(ln(x)) + 2cos(ln(x)))

The equation is satisfied for y₂.

Therefore, the solutions to the given differential equations are:

y = x³ + 7

y = 3e^(-2x)

y₁ = cos(2x)

y₁ = e^(3x), y₂ = e^(-3x)

y = e^x - e^(-x)

y₁ = e^(-2x)

y₁ = e^x cos(x), y₂ = e^x sin(x)

y = 1 + x²

y₁ = xcos(ln(x)), y₂ = xsin(ln(x))

Learn more about differential equation

https://brainly.com/question/32538700

#SPJ11

[4 marks] Prove that the number √7 lies between 2 and 3. Question 3.[4 marks] Fix a constant r> 1. Using the Mean Value Theorem prove that ez > 1 + rr

Answers

Question 1

We know that √7 can be expressed as 2.64575131106.

Now, we need to show that this number lies between 2 and 3.2 < √7 < 3

Let's square all three numbers.

We get; 4 < 7 < 9

Since the square of 2 is 4, and the square of 3 is 9, we can conclude that 2 < √7 < 3.

Hence, the number √7 lies between 2 and 3.

Question 2

Let f(x) = ez be a function.

We want to show that ez > 1 + r.

Using the Mean Value Theorem (MVT), we can prove this.

The statement of the MVT is as follows:

If a function f(x) is continuous on the closed interval [a, b] and differentiable on the open interval (a, b), then there exists a point c in the interval (a, b) such that

f'(c) = [f(b) - f(a)]/[b - a].

Now, let's find f'(x) for our function.

We know that the derivative of ez is ez itself.

Therefore, f'(x) = ez.

Then, let's apply the MVT.

We have

f'(c) = [f(b) - f(a)]/[b - a]

[tex]e^c = [e^r - e^1]/[r - 1][/tex]

Now, we have to show that [tex]e^r > 1 + re^(r-1)[/tex]

By multiplying both sides by (r-1), we get;

[tex](r - 1)e^r > (r - 1) + re^(r-1)e^r - re^(r-1) > 1[/tex]

Now, let's set g(x) = xe^x - e^(x-1).

This is a function that is differentiable for all values of x.

We know that g(1) = 0.

Our goal is to show that g(r) > 0.

Using the Mean Value Theorem, we have

g(r) - g(1) = g'(c)(r-1)

[tex]e^c - e^(c-1)[/tex]= 0

This implies that

[tex](r-1)e^c = e^(c-1)[/tex]

Therefore,

g(r) - g(1) = [tex](e^(c-1))(re^c - 1)[/tex]

> 0

Thus, we have shown that g(r) > 0.

This implies that [tex]e^r - re^(r-1) > 1[/tex], as we had to prove.

To know more about Mean Value Theorem   visit:

https://brainly.com/question/30403137

#SPJ11

For each natural number n and each number x in (-1, 1), define f₁(x)=√√√x² √ x² + = ₁ and define f(x) = |x|. Prove that the sequence (ƒ: (-1, 1)→ R} converges uni- formly to the function f: (-1, 1)→ R. Check that each function f: (-1, 1)→ Ris differentiable, whereas the limit function ƒ: (−1, 1) → R is hot differentiable. Does this contradict Theorem 9.19? Thm Let I be an open interval. Suppose that (f: I→ R) is a sequence of continuously differentiable functions that has the following two properties: 9.19. (i) The sequence {f: 1 → R} converges pointwise to the function f: 1 → R and (ii) The sequence of derivatives {f:I→ R} converges uniformly to the function 8:1 → R. Then the function f:I → R is continuously differentiable and f'(x) = g(x) for all x in [a, b].

Answers

In this problem, we are given two sequences of functions: f₁(x) = √√√x² √ x² + and f(x) = |x|. We need to prove that the sequence (ƒ: (-1, 1)→ R} converges uniformly to the function f: (-1, 1)→ R.

We also need to check the differentiability of each function and observe that the limit function ƒ: (−1, 1) → R is not differentiable.

We then consider whether this contradicts Theorem 9.19, which states conditions for the continuity of the derivative.

To prove that the sequence (ƒ: (-1, 1)→ R} converges uniformly to the function f: (-1, 1)→ R, we need to show that for any ε > 0, there exists an N such that for all x in (-1, 1) and n > N, |ƒₙ(x) - ƒ(x)| < ε.

This can be done by analyzing the behavior of the two sequences f₁(x) and f(x), and showing that their values converge to the same function f(x) = |x| uniformly.

Next, we check the differentiability of each function. The function f₁(x) = √√√x² √ x² + is continuously differentiable for all x in (-1, 1) since it is a composition of continuous functions.

The function f(x) = |x| is not differentiable at x = 0 because it has a sharp corner or "kink" at that point.

This observation leads us to the fact that the limit function ƒ(x) = |x| is also not differentiable at x = 0.

This does not contradict Theorem 9.19 because the conditions of the theorem require the sequence of derivatives {fₙ'(x)} to converge uniformly to the derivative function g(x).

In this case, the sequence of derivatives does not converge uniformly since the derivative of fₙ(x) is not defined at x = 0, while the derivative of f(x) exists and is equal to ±1 depending on the sign of x.

Therefore, the fact that the limit function ƒ(x) = |x| is not differentiable at x = 0 does not contradict Theorem 9.19 because the conditions of the theorem are not satisfied.

In conclusion, the sequence (ƒ: (-1, 1)→ R} converges uniformly to the function f: (-1, 1)→ R, each function f: (-1, 1)→ R is differentiable except for the limit function, and this observation does not contradict Theorem 9.19.

To learn more about sequence visit:

brainly.com/question/23857849

#SPJ11

x(2x-4) =5 is in standard form

Answers

Answer:
[tex]2x^2-4x-5=0[/tex] is standard form.

Step-by-step explanation:
Standard form of a quadratic equation should be equal to 0. Standard form should be [tex]ax^2+bx+c=0[/tex], unless this isn't a quadratic equation?

We can convert your equation to standard form with a few calculations. First, subtract 5 from both sides:

[tex]x(2x-4)-5=0[/tex]

Then, distribute the x in front:

[tex]2x^2-4x-5=0[/tex]

The equation should now be in standard form. (Unless, again, this isn't a quadratic equation – "standard form" can mean different things in different areas of math).

he answer above is NOT correct. (1 point) A street light is at the top of a 18 foot tall pole. A 6 foot tall woman walks away from the pole with a speed of 4 ft/sec along a straight path. How fast is the tip of her shadow moving when she is 45 feet from the base of the pole? The tip of the shadow is moving at 2 ft/sec.

Answers

The tip of the woman's shadow is moving at a rate of 2 ft/sec when she is 45 feet from the base of the pole, confirming the given information.

Let's consider the situation and set up a right triangle. The height of the pole is 18 feet, and the height of the woman is 6 feet. As the woman walks away from the pole, her shadow is cast on the ground, forming a similar triangle with the pole. Let the length of the shadow be x.

By similar triangles, we have the proportion: (6 / 18) = (x / (x + 45)). Solving for x, we find that x = 15. Therefore, when the woman is 45 feet from the base of the pole, her shadow has a length of 15 feet.

To find the rate at which the tip of the shadow is moving, we can differentiate the above equation with respect to time: (6 / 18) dx/dt = (x / (x + 45)) d(x + 45)/dt. Plugging in the given values, we have (2 / 3) dx/dt = (15 / 60) d(45)/dt. Solving for dx/dt, we find that dx/dt = (2 / 3) * (15 / 60) * 2 = 2 ft/sec.

Learn more about right triangle here:

https://brainly.com/question/29285631

#SPJ11

Find the volume of the solid generated by revolving the region bounded by the graphs of the equations about the line y 5. (Round your answer to three decimal places) 4 Y= 1+x y=0 x=0 X-4

Answers

The volume of solid generated by revolving the region bounded by the graphs of the equations about the line y = 5 is ≈ 39.274 cubic units (rounded to three decimal places).

We are required to find the volume of the solid generated by revolving the region bounded by the graphs of the equations about the line y = 5.

We know the following equations:

y = 0x = 0

y = 1 + xx - 4

Now, let's draw the graph for the given equations and region bounded by them.

This is how the graph would look like:

graph{y = 1+x [-10, 10, -5, 5]}

Now, we will use the Disk Method to find the volume of the solid generated by revolving the region bounded by the graphs of the equations about the line y = 5.

The formula for the disk method is as follows:

V = π ∫ [R(x)]² - [r(x)]² dx

Where,R(x) is the outer radius and r(x) is the inner radius.

Let's determine the outer radius (R) and inner radius (r):

Outer radius (R) = 5 - y

Inner radius (r) = 5 - (1 + x)

Now, the volume of the solid generated by revolving the region bounded by the graphs of the equations about the line y = 5 is given by:

V = π ∫ [5 - y]² - [5 - (1 + x)]² dx

= π ∫ [4 - y - x]² - 16 dx  

[Note: Substitute (5 - y) = z]

Now, we will integrate the above equation to find the volume:

V = π [ ∫ (16 - 8y + y² + 32x - 8xy - 2x²) dx ]

(evaluated from 0 to 4)

V = π [ 48√2 - 64/3 ]

≈ 39.274

Know more about the solid generated

https://brainly.com/question/32493136

#SPJ11

For f(x)=√x and g(x) = 2x + 3, find the following composite functions and state the domain of each. (a) fog (b) gof (c) fof (d) gog (a) (fog)(x) = (Simplify your answer.) For f(x) = x² and g(x)=x² + 1, find the following composite functions and state the domain of each. (a) fog (b) gof (c) fof (d) gog (a) (fog)(x) = (Simplify your answer.) For f(x) = 5x + 3 and g(x)=x², find the following composite functions and state the domain of each. (a) fog (b) gof (c) fof (d) gog (a) (fog)(x) = (Simplify your answer.)

Answers

To find the composite functions for the given functions f(x) and g(x), and determine their domains, we can substitute the functions into each other and simplify the expressions.

(a) For (fog)(x):

Substituting g(x) into f(x), we have (fog)(x) = f(g(x)) = f(2x + 3) = √(2x + 3).

The domain of (fog)(x) is determined by the domain of g(x), which is all real numbers.

Therefore, the domain of (fog)(x) is also all real numbers.

(b) For (gof)(x):

Substituting f(x) into g(x), we have (gof)(x) = g(f(x)) = g(√x) = (2√x + 3).

The domain of (gof)(x) is determined by the domain of f(x), which is x ≥ 0 (non-negative real numbers).

Therefore, the domain of (gof)(x) is x ≥ 0.

(c) For (fof)(x):

Substituting f(x) into itself, we have (fof)(x) = f(f(x)) = f(√x) = √(√x) = (x^(1/4)).

The domain of (fof)(x) is determined by the domain of f(x), which is x ≥ 0.

Therefore, the domain of (fof)(x) is x ≥ 0.

(d) For (gog)(x):

Substituting g(x) into itself, we have (gog)(x) = g(g(x)) = g(2x + 3) = (2(2x + 3) + 3) = (4x + 9).

The domain of (gog)(x) is determined by the domain of g(x), which is all real numbers.

Therefore, the domain of (gog)(x) is also all real numbers.

In conclusion, the composite functions and their domains are as follows:

(a) (fog)(x) = √(2x + 3), domain: all real numbers.

(b) (gof)(x) = 2√x + 3, domain: x ≥ 0.

(c) (fof)(x) = x^(1/4), domain: x ≥ 0.

(d) (gog)(x) = 4x + 9, domain: all real numbers.

Learn more about functions here: brainly.com/question/30660139

#SPJ11

Find general solution for the ODE 9x y" - gy e3x =

Answers

The general solution of the given ODE 9x y" - gy e3x = 0 is given by y(x) = [(-1/3x) + C1] * 1 - [(1/9x) - (1/81) + C2] * (g/27) * e^(3x).

To find general solution of the ODE:

Step 1: Finding the first derivative of y

Wrtie the given equation in the standard form as:

y" - (g/9x) * e^(3x) * y = 0

Compare this with the standard form of the homogeneous linear ODE:

y" + p(x) y' + q(x) y = 0, we have

p(x) = 0q(x) = -(g/9x) * e^(3x)

Integrating factor (IF) of this ODE is given by:

IF = e^∫p(x)dx = e^∫0dx = 1

Therefore, multiplying both sides of the ODE by the integrating factor, we have:

y" + (g/9x) * e^(3x) * y' = 0 …….(1)

Step 2: Using the Method of Variation of Parameters to find the general solution of the ODE. Assuming the solution of the form

y = u1(x) y1(x) + u2(x) y2(x),

where y1 and y2 are linearly independent solutions of the homogeneous ODE (1).

So, y1 = 1 and y2 = ∫q(x) / y1^2(x) dx

Solving the above expression, we get:

y2 = ∫[-(g/9x) * e^(3x)] dx = -(g/27) * e^(3x)

Taking y1 = 1 and y2 = -(g/27) * e^(3x)

Now, using the formula for the method of variation of parameters, we have

u1(x) = (- ∫y2(x) f(x) dx) / W(y1, y2)

u2(x) = ( ∫y1(x) f(x) dx) / W(y1, y2),

where W(y1, y2) is the Wronskian of y1 and y2.

W(y1, y2) = |y1 y2' - y1' y2|

= |1 (-g/9x) * e^(3x) + 0 g/3 * e^(3x)|

= g/9x^2 * e^(3x)So,u1(x)

= (- ∫[-(g/27) * e^(3x)] (g/9x) * e^(3x) dx) / (g/9x^2 * e^(3x))

= (-1/3x) + C1u2(x)

= ( ∫1 (g/9x) * e^(3x) dx) / (g/9x^2 * e^(3x))

= [(1/3x) - (1/27)] + C2

where C1 and C2 are constants of integration.

Therefore, the general solution of the given ODE is

y(x) = u1(x) y1(x) + u2(x) y2(x)y(x) = [(-1/3x) + C1] * 1 - [(1/9x) - (1/81) + C2] * (g/27) * e^(3x)

Learn more about derivative visit:

brainly.com/question/29144258

#SPJ

how to change a negative exponent to a positive exponent

Answers

Here’s an example
X^-2 = 1/x^2

Is y= x+6 a inverse variation

Answers

Answer:

No, y = x  6 is not an inverse variation

Step-by-step explanation:

In Maths, inverse variation is the relationships between variables that are represented in the form of y = k/x, where x and y are two variables and k is the constant value. It states if the value of one quantity increases, then the value of the other quantity decreases.

No, y = x + 6 is not an inverse variation. An inverse variation is a relationship between two variables in which their product is a constant. In other words, as one variable increases, the other variable decreases in proportion to keep the product constant. The equation of an inverse variation is of the form y = k/x, where k is a constant. In the equation y = x + 6, there is no inverse relationship between x and y, as there is no constant k that can be multiplied by x to obtain y. Therefore, it is not an inverse variation.

A building worth $835,000 is depreciated for tax purposes by its owner using the straight-line depreciation method. The value of the building, y, after x months of use. is given by y 835,000-2300x dollars. After how many years will the value of the building be $641,8007 The value of the building will be $641,800 after years. (Simplify your answer. Type an integer or a decimal)

Answers

It will take approximately 7 years for the value of the building to be $641,800.

To find the number of years it takes for the value of the building to reach $641,800, we need to set up the equation:

835,000 - 2,300x = 641,800

Let's solve this equation to find the value of x:

835,000 - 2,300x = 641,800

Subtract 835,000 from both sides:

-2,300x = 641,800 - 835,000

-2,300x = -193,200

Divide both sides by -2,300 to solve for x:

x = -193,200 / -2,300

x ≈ 84

Therefore, it will take approximately 84 months for the value of the building to reach $641,800.

To convert this to years, divide 84 months by 12:

84 / 12 = 7

Hence, it will take approximately 7 years for the value of the building to be $641,800.

To know more about the equation visit:

https://brainly.com/question/10416394

#SPJ11

Find solutions for your homework
Find solutions for your homework
mathcalculuscalculus questions and answersmy notes ask your teacher given f(x) = -7 + x2, calculate the average rate of change on each of the given intervals. (a) the average rate of change of f(x) over the interval [-6, -5.9] is (b) the average rate of change of f(x) over the interval [-6, -5.99] is (c) the average rate of change of f(x) over the interval [-6, -5.999] is (d) using (a) through (c)
This problem has been solved!
You'll get a detailed solution from a subject matter expert that helps you learn core concepts.
See Answer
Question: MY NOTES ASK YOUR TEACHER Given F(X) = -7 + X2, Calculate The Average Rate Of Change On Each Of The Given Intervals. (A) The Average Rate Of Change Of F(X) Over The Interval [-6, -5.9] Is (B) The Average Rate Of Change Of F(X) Over The Interval [-6, -5.99] Is (C) The Average Rate Of Change Of F(X) Over The Interval [-6, -5.999] Is (D) Using (A) Through (C)
MY NOTES
ASK YOUR TEACHER
Given f(x) = -7 + x2, calculate the average rate of change on each of the given intervals.
(a) The
Show transcribed image text
Expert Answer
answer image blur
Transcribed image text: MY NOTES ASK YOUR TEACHER Given f(x) = -7 + x2, calculate the average rate of change on each of the given intervals. (a) The average rate of change of f(x) over the interval [-6, -5.9] is (b) The average rate of change of f(x) over the interval [-6, -5.99] is (c) The average rate of change of f(x) over the interval [-6, -5.999] is (d) Using (a) through (c) to estimate the instantaneous rate of change of f(x) at x = -6, we have Submit Answer 2. [-/0.76 Points] DETAILS TAMUBUSCALC1 2.1.002. 0/6 Submissions Used MY NOTES ASK YOUR TEACHER For the function y 9x2, find the following. (a) the average rate of change of f(x) over the interval [1,4] (b) the instantaneous rate of change of f(x) at the value x = 1

Answers

The average rate of change of f(x) over the interval [-6, -5.9] is 13.9, the average rate of change of f(x) over the interval [-6, -5.99] is 3.99, the average rate of change of f(x) over the interval [-6, -5.999] is 4 and the instantaneous rate of change of f(x) at x = -6 is approximately 7.3.

Given the function

f(x) = -7 + x²,

calculate the average rate of change on each of the given intervals.

Interval -6 to -5.9:

This interval has a length of 0.1.

f(-6) = -7 + 6²

= 19

f(-5.9) = -7 + 5.9²

≈ 17.61

The average rate of change of f(x) over the interval [-6, -5.9] is:

(f(-5.9) - f(-6))/(5.9 - 6)

= (17.61 - 19)/(-0.1)

= 13.9

Interval -6 to -5.99:

This interval has a length of 0.01.

f(-5.99) = -7 + 5.99²

≈ 18.9601

The average rate of change of f(x) over the interval [-6, -5.99] is:

(f(-5.99) - f(-6))/(5.99 - 6)

= (18.9601 - 19)/(-0.01)

= 3.99

Interval -6 to -5.999:

This interval has a length of 0.001.

f(-5.999) = -7 + 5.999²

≈ 18.996001

The average rate of change of f(x) over the interval [-6, -5.999] is:

(f(-5.999) - f(-6))/(5.999 - 6)

= (18.996001 - 19)/(-0.001)

= 4

Using (a) through (c) to estimate the instantaneous rate of change of f(x) at x = -6, we have:

[f'(-6) ≈ 13.9 + 3.99 + 4}/{3}

= 7.3

Know more about the average rate of change

https://brainly.com/question/8728504

#SPJ11

Recently, a certain bank offered a 10-year CD that earns 2.83% compounded continuously. Use the given information to answer the questions. (a) If $30,000 is invested in this CD, how much will it be worth in 10 years? approximately $ (Round to the nearest cent.) (b) How long will it take for the account to be worth $75,000? approximately years (Round to two decimal places as needed.)

Answers

If $30,000 is invested in a CD that earns 2.83% compounded continuously, it will be worth approximately $43,353.44 in 10 years. It will take approximately 17.63 years for the account to reach $75,000.

To solve this problem, we can use the formula for compound interest:

```

A = P * e^rt

```

where:

* A is the future value of the investment

* P is the principal amount invested

* r is the interest rate

* t is the number of years

In this case, we have:

* P = $30,000

* r = 0.0283

* t = 10 years

Substituting these values into the formula, we get:

```

A = 30000 * e^(0.0283 * 10)

```

```

A = $43,353.44

```

This means that if $30,000 is invested in a CD that earns 2.83% compounded continuously, it will be worth approximately $43,353.44 in 10 years.

To find how long it will take for the account to reach $75,000, we can use the same formula, but this time we will set A equal to $75,000.

```

75000 = 30000 * e^(0.0283 * t)

```

```

2.5 = e^(0.0283 * t)

```

```

ln(2.5) = 0.0283 * t

```

```

t = ln(2.5) / 0.0283

```

```

t = 17.63 years

```

This means that it will take approximately 17.63 years for the account to reach $75,000.

Learn more about compound interest here:

brainly.com/question/14295570

#SPJ11

Find all lattice points of f(x)=log3(x+1)−9

Answers

Answer:

Step-by-step explanation:

?

point ;)

Find the indefinite integral using partial fractions. √² 2z²+91-9 1³-31² dz

Answers

To find the indefinite integral using partial fractions of √(2z^2 + 91)/(1 - 31z^2) dz, we need to first factorize the denominator and then decompose the fraction into partial fractions.

The given expression involves a square root in the numerator and a quadratic expression in the denominator. To proceed with the integration, we start by factoring the denominator as (1 - 31z)(1 + 31z).

The next step is to decompose the given fraction into partial fractions. Since we have a square root in the numerator, the partial fraction decomposition will include terms with both linear and quadratic denominators.

Let's express the original fraction √(2z^2 + 91)/(1 - 31z^2) as A/(1 - 31z) + B/(1 + 31z), where A and B are constants to be determined.

To find the values of A and B, we multiply both sides of the equation by the denominator (1 - 31z^2) and simplify:

√(2z^2 + 91) = A(1 + 31z) + B(1 - 31z)

Squaring both sides of the equation to remove the square root:

2z^2 + 91 = (A^2 + B^2) + 31z(A - B) + 62Az

Now, we equate the coefficients of like terms on both sides of the equation:

Coefficient of z^2: 2 = A^2 + B^2

Coefficient of z: 0 = 31(A - B) + 62A

Constant term: 91 = A^2 + B^2

From the second equation, we have:

31A - 31B + 62A = 0

93A - 31B = 0

93A = 31B

Substituting this into the first equation:

2 = A^2 + (93A/31)^2

2 = A^2 + 3A^2

5A^2 = 2

A^2 = 2/5

A = ±√(2/5)

Since A = ±√(2/5) and 93A = 31B, we can solve for B:

93(±√(2/5)) = 31B

B = ±3√(2/5)

Therefore, the partial fraction decomposition is:

√(2z^2 + 91)/(1 - 31z^2) = (√(2/5)/(1 - 31z)) + (-√(2/5)/(1 + 31z))

Now we can integrate each partial fraction separately:

∫(√(2/5)/(1 - 31z)) dz = (√(2/5)/31) * ln|1 - 31z| + C1

∫(-√(2/5)/(1 + 31z)) dz = (-√(2/5)/31) * ln|1 + 31z| + C2

Where C1 and C2 are integration constants.

Thus, the indefinite integral using partial fractions is:

(√(2/5)/31) * ln|1 - 31z| - (√(2/5)/31) * ln|1 + 31z| + C, where C = C1 - C2.

To learn more about integral  Click Here: brainly.com/question/31433890

#SPJ11

Say we have some closed set B that is a subset of R, B has some suprema sup B. Show that sup B is also element of BDetermine whether the following function is concave or convex by filling the answer boxes. f(x)=x-x² *** By using the definition of concave function we have the following. f(ha+(1-x)b) ≥f(a) + (1 -λ)f(b) with a, b in the domain of f and XE[0, 1], we have that ha+(1-A)b-[ha+(1-2)b]² ≥ (a-a²)+ Simplifying and rearranging the terms leads to [Aa +(1-2)b]2a² + (1 -λ)b² Moving all the terms to the left hand side of the inequality and simplifying leads to SO This inequality is clearly respected and therefore the function is

Answers

In this case, since f''(x) = -2 < 0 for all x in the domain of f(x) = x - x², the function is concave.

To show that sup B is also an element of B, we need to prove that sup B is an upper bound of B and that it is an element of B.

Upper Bound: Let b be any element of B. Since sup B is the least upper bound of B, we have b ≤ sup B for all b in B. This shows that sup B is an upper bound of B.

Element of B: We need to show that sup B is also an element of B. Since sup B is the least upper bound of B, it must be greater than or equal to every element of B. Therefore, sup B ≥ b for all b in B, including sup B itself. This shows that sup B is an element of B.

Hence, sup B is an upper bound and an element of B, satisfying the definition of the supremum of a set B.

Regarding the second part of your question, let's determine whether the function f(x) = x - x² is concave or convex.

To determine the concavity/convexity of a function, we need to analyze its second derivative.

First, let's find the first derivative of f(x):

f'(x) = 1 - 2x

Now, let's find the second derivative:

f''(x) = -2

Since the second derivative f''(x) = -2 is a constant, we can determine the concavity/convexity based on its sign.

If f''(x) < 0 for all x in the domain, then the function is concave.

If f''(x) > 0 for all x in the domain, then the function is convex.

To know more about function,

https://brainly.com/question/29397735

#SPJ11

The graph below represents a map of the distance from Blake's house to the school

If each unit on the graph represents 0.75 miles, how many miles is the diagonal path from Blake's house to the school?


HELP!! 100 Brainly points given!!

Answers

Answer:

C. 6 miles

Step-by-step explanation:

If each unit on the graph is 0.75 miles that means each box is 0.75 miles.

So you must count how many boxes it takes to reach the school from Blake's house. Count the amount of boxes the line passes through.

So in this case 8 boxes are crossed to get to the school.

Therefore you do:

8 × 0.75 = 6

Answer = 6 miles

By selling 12 apples for a rupee,a man loses 20% .How many for a rupee should be sold to gain 20%​

Answers

Answer: The selling price of 8 apples for a rupee will give a 20% profit.

Step-by-step explanation: To find the cost price of each apple, you can use the formula: Cost price = Selling price / Quantity. To find the selling price that will give a 20% profit, use the formula: Selling price = Cost price + Profit.

- Lizzy ˚ʚ♡ɞ˚

The difference is five: Help me solve this View an example Ge This course (MGF 1107-67404) is based on Angel:

Answers

The difference is 13₅.

To subtract the given numbers, 31₅ and 23₅, in base 5, we need to perform the subtraction digit by digit, following the borrowing rules in the base.

Starting from the rightmost digit, we subtract 3 from 1. Since 3 is larger than 1, we need to borrow from the next digit. In base 5, borrowing 1 means subtracting 5 from 11. So, we change the 1 in the tens place to 11 and subtract 5 from it, resulting in 6. Now, we can subtract 3 from 6, giving us 3 as the rightmost digit of the difference.

Moving to the left, there are no digits to borrow from in this case. Therefore, we can directly subtract 2 from 3, giving us 1.

Therefore, the difference of 31₅ - 23₅ is 13₅.

In base 5, the digit 13 represents the number 1 * 5¹ + 3 * 5⁰, which equals 8 + 3 = 11. Therefore, the difference is 11 in base 10.

In conclusion, the difference of 31₅ - 23₅ is 13₅ or 11 in base 10.

Correct Question :

Subtract The Given Numbers In The Indicated Base. 31_five - 23_five.

To learn more about difference here:

https://brainly.com/question/28808877

#SPJ4

The sequence {an} is monotonically decreasing while the sequence {b} is monotonically increasing. In order to show that both {a} and {bn} converge, we need to confirm that an is bounded from below while br, is bounded from above. Both an and b, are bounded from below only. an is bounded from above while bn, is bounded from below. Both and b, are bounded from above only. O No correct answer is present. 0.2 pts

Answers

To show that both the sequences {a} and {bn} converge, it is necessary to confirm that an is bounded from below while bn is bounded from above.

In order for a sequence to converge, it must be both monotonic (either increasing or decreasing) and bounded. In this case, we are given that {an} is monotonically decreasing and {b} is monotonically increasing.

To prove that {an} converges, we need to show that it is bounded from below. This means that there exists a value M such that an ≥ M for all n. Since {an} is monotonically decreasing, it implies that the sequence is bounded from above as well. Therefore, an is both bounded from above and below.

Similarly, to prove that {bn} converges, we need to show that it is bounded from above. This means that there exists a value N such that bn ≤ N for all n. Since {bn} is monotonically increasing, it implies that the sequence is bounded from below as well. Therefore, bn is both bounded from below and above.

In conclusion, to establish the convergence of both {a} and {bn}, it is necessary to confirm that an is bounded from below while bn is bounded from above.

Learn more about convergence of a sequence:

https://brainly.com/question/29394831

#SPJ11

Other Questions
2. List four industries that are monopolisticaly competitive. What percentage of industry output is produced by each of the four largest firms, p 252/258.3. Graphically illustrate short run profit & loss plus long run equilibrium for a monoplisticaly competitive firm, p.254/260.6. List & explain three characteristics of oligopoly, p. 261 to p. 262/268.please answer thus three questions And mention the number which one for which Macroeconomics Group of answer choices is the efficient allocation of societies scarce resources is how to use our scarce resources to maximize societies well being is studied so society can use its limited resources to its maximum potential and increase well being All of the above As an executive with responsibility for new product development, a subordinate has just placed on your desk a copy of a fancy-looking "product space map" to helpsupport his argument in favor of developing and introducing a new product. What key questions should you ask about how this map was generated, what it assumes, and how it should be interpreted before you attempt to use the map as the basis for any decision-making. Suppose meat producers create a negative externality. Also, suppose that the government imposes a tax on the producers equal to the per-unit externality. What is the relationship between the equilibrium quantity and the quantity that should be produced? A) They are equal. B) The equilibrium quantity is greater than what should be produced C) The equilibrium quantity is less than what should be produced D) Not enough information to answer the question anomie theory is sometimes referred to as what other theory? Using a suitable linearization to approximate 101, show that (i) The approximate value is 10.05. (ii) The error is at most = 0.00025. That is 101 (10.04975, 10.05025). 4000 Diagonalization 8. Diagonalize A= [$] 11 9 3 9. Diagonalize A = 6 14 3 -36-54-13 5 -8 10. Orthogonally diagonalize. -8 5 4 -4 -1 11. Let Q(,. 3) = 5x-16122+81+5-813-23, 12, 13 R. Find the maximum and minimum value of Q with the constraint a++=1. Part IV Inner Product 12. Find a nonzero vector which is orthogonal to the vectors = (1,0,-2) and (1,2,-1). 13. If A and B are arbitrary real mx n matrices, then the mapping (A, B) trace(ATB) defines an inner product in RX, Use this inner product to find (A, B), the norms ||A|| and B, and the angle og between A and B for -3 1 2 and B= 22 ----B -1 -2 2 14. Find the orthogonal projection of -1 14 7 = -16 12 onto the subspace W of R spanned by and 2 -18 15. Find the least-squares solution of the system B-E 7= 16. By using the method of least squares, find the best parabola through the points: (1, 2), (2,3), (0,3), (-1,2) Suppose Show that 1.2 Show that if || = 1, then = a + ib and = a + ib. 2132 = (51) (5). 2 +22+6+8i| 13. (5) (5) Suppose that you have the following information about aperfectly competitive firm:P= $8; Q= 1000; ATC= $9; AVC= $7.8; MC= $7Based on this information, answer the following questions.Calculate the amount of profit the firm is currently making, firms current producer surplus, explain if the firm should stay in business or shut down, and can the firm increase profit by changing output level explain and show your working. Solve using Laplace Transforms. (a) y" - 3y + 2y = e; 1 Solution: y = = + 6 (b) x'- 6x + 3y = 8et y' - 2xy = 4et x (0) = -1 y (0) = 0 2 Solution: x(t) = e4 2e', y(t) = -e4. 3 y(0) = 1, y'(0) = 0 3 Zez 2 22 2 COIN It is important that energy needs in pregnancy are met so that:1.optimal fetal growth is ensured.2.adequate blood volume is provided.3.maternal vitamins are spared.4.adipose tissue stores are spared. Jasa Tiasa Berhad Holdings Berhad is the preferred quality oil palm and timber producer in Malaysia. Currently, the company announces to sell a three-month call option. The share price is RM50 and the strike price is RM45. The risk-free rate is 8 percent per annum and the volatility is 35 percent. Calculate the value of call option. A curve C is defined by the parametric equations r = 3t, y = 5t-t. (a) Find all of the points on C where the tangents is horizontal or vertical. (b) Find the two equations of tangents to C at (,0). (c) Determine where the curve is concave upward or downward. Disseminated by Kodak to labs that processed film, the Shirley card was a reference card used to calibrate skin tone. The original Shirley card, named for the first woman who sat for it, and many subsequent versions, used only one model in the image, and she was white. This lasted until 1995, when Kodak introduced a reference card featuring a white, Asian, and black woman with different skin tones. The Shirley card serves as an example of the ways in which ____________ are embedded in machine design, put into play just by using them.A. countervisual strategiesB. social and political perspectivesC. algorithmsD. technological advancements Given the Linear Optimization Problem: min (x1 4x2 3x3)2x1 + 2x2 + x3 4x1 + 2x2 + 2x3 6x1, x2, x3 0State the dual problem. What is the optimal value for the primal and the dual? What is the duality gap?Expert AnswerSolution for primal Now convert primal problem to DView the full answeranswer image blurPrevious questionNext question ABC Company's variable cost ratio is 75% and the break-even point in sales dollars is $200,000. If ABC Company reported a net income of $60,000, sales revenue must have been equal to: $280,000 $420,000 $200,000 $480,000 $260,000 $440,000 Identify the lotter for the principle or assumption from A through D in the blank space next to each numbered situation that it best explains or justifies. _____ In proparing financial statements for Dockside Digs, the accountant makes sure that the expense transactions of the owner are kept separate from the company's iransactions and financial statements. _____ When Ahmed clinic buys medical equipment, provides a health service, or uses an Eaverue recognitien assumption asset, they record the monetary value of these transactions. ______ In December 2022 of this year, Chavez construction recelved a customer's order and cash prepayment to build a house that would not be ready until March 2023 . Chavez should rocord the rovenue from the customer order in March 2023, fot in December 2022. _____ Rasheed Sottware classifies assets and liabilities in the balance sheet into carrent and noncurrent to refiect the fact that the business will continue operating for the foreseeable future.A. Business entity assumptionB. Monetary value assumptionD. Going concem assumption what is the inverse of the given function? y = 3x + 9 Independent random samples, each containing 700 observations, were selected from two binomial populations. The samples from populations 1 and 2 produced 690 and 472 successes, respectively.(a) Test H0:(p1p2)=0 against Ha:(p1p2)0. Use =0.07test statistic =rejection region |z|>The final conclusion is Chile and Argentina produce jellybeans (x) and peanut botter (y) using labot as their only resources. Each country has a 1000 hours and Chile uses 1 hour to produce jellybeans and 2 hours to produce peamut butter. Argentina uses 1 hour to produce jellybeans and 4 hours to produce peanur butter Plot the PPFs for both countres Chale and Argentins and B. Write the pre-tnde price fatio in each country and comparel. Tabel the pre trube or autashy consumption/production point with no bste bias, anternational paice ratio, pest thace prodactios and consamption and the trade triangle!What is the basis fot trade in thas model? Can these countries completely specialixe or not? _____ Explain why of whey not? ____The word peodaction of good X and Y before thade X= _____. Y= _____. The world pcoduction of good X and Y after trade X= _____. Y= _____. How do you show the gaans from trader?